The selling price for peanut butter is $3. 79 and the cost is $2. 84

Answers

Answer 1

Answer: Profit = .95 or 95 cents

Step-by-step explanation:

What is the question here?

If it is profit then the answer is 3.79-2.84


Related Questions

44 friends evenly divided up an

n-slice pizza. One of the friends, Harris, ate
1
11 fewer slices than he received.
how many slices did he eat

Answers

The expression for the number of slices Harris ate is [tex](n/44) - 11.[/tex]

What is the expression for slices eaten?

Let s be the number of slices each friend received.

Then the total number of slices on the pizza is n and we have: 44s = n because pizza was evenly divided among the 44 friends). Harris ate 11 fewer slices than he received, so he ate s - 11 slices.

The no of slices he  received is s and we have:

s = (s - 11) + Harris's slices eaten

Substituting 44s = n, we will solve for s:

44s = n

s = n/44

So, the expression for the number of slices Harris ate is [tex](n/44) - 11.[/tex]

Read more about expression

brainly.com/question/1859113

#SPJ1

7x-6° 120° Find the value of x

Answers

Answer:

x = 18°

Step-by-step explanation:

7x - 6° = 120°

7x = 126°

x = 18°

So, the answer is x = 18°

Javae got $100 for his birthday. He listened to a Podcast about investing, and decided he would try it. He invested his $100 into a fund that has 9% growth per year. How much will Javae have in that account on his 30th birthday? (16 years
from now).

O 1,327
O $22
O $2,884,414
O $397

Answers

Answer:

The formula for compound interest is A = P(1 + r/n)^(nt), where A is the final amount, P is the initial principal, r is the interest rate, n is the number of times the interest is compounded per year, and t is the time in years.

In this case, P = $100, r = 9% = 0.09, n = 1 (compounded annually), and t = 16. We want to find A.

A = 100(1 + 0.09/1)^(1*16)

A = $397.42

Therefore, Javae will have $397.42 in his account on his 30th birthday. The answer is option D.

A pitcher contains 3 quarts of iced
tea. John is going to pour the iced tea
into glasses that each hold 1½ cups.
How many glasses will John fill?

Answers

John will be able to fill 8 glasses with iced tea.

How many cups are in 3 quarts of iced tea?

To convert quarts to cups, we need to multiply the number of quarts by 4, since 1 quart is equal to 4 cups. So, 3 quarts of iced tea is equivalent to 3 x 4 = 12 cups of iced tea.

Since each glass holds 1½ cups of iced tea, John will need to divide the total cups of iced tea by the capacity of each glass: 12 cups / 1.5 cups/glass = 8 glasses.

Therefore, John will fill 8 glasses with iced tea.

Read more about iced tea

brainly.com/question/30179276

#SPJ1

A deck of standard playing cards holds 52 unique cards.
36 of these cards are numbered cards (numbered 2-9), 4
of these cards are aces, and 12 of these cards are face
cards (jack, queen, and king). If you play a card game and
draw half of the face cards, one ace, and one fourth of the
numbered cards, how many cards do you have? How
many of each card type of card do you have? Cite
evidence from the problem.

PLS HELP

Answers

The total number of cards drawn is 16 and As a result, we have the following:

6 face cards, Aces 1, 9 numbered cards

How to determine the type of each card?

Because there are 12 face cards, half of them are 6 face cards.

There are 4 aces, so we draw 1.

There are 36 numbered cards in total, therefore one-fourth of them are 9 numbered cards.

We draw a total of 6 + 1 + 9 = 16 cards.

We may use the information provided in the problem to determine how many of each card type we have. We chose:

6 face cards make up half of the deck.

1 ace: 1 ace

9 numbered cards are one-fourth of the numbered cards.

As a result, we have:

6 face cards

Aces: 1

9 numbered cards

We can also ensure that the total number of cards drawn (16) corresponds to the number of cards in a regular deck by subtracting the cards we drew from the total number of cards: a deck of playing cards (52)

There are 52 - 16 = 36 cards left.

This means that there are 36 - 6 - 1 - 9 = 20 undrawn cards in the deck.

Learn more about cards here:

https://brainly.com/question/29165788

#SPJ1

Monte owns a bakery. He has four employees, who each earn a different hourly wage.
Which two quantities are needed to determine the total amount of money Monte paid his employees this month?
A. the hourly salary of each employee
B. the number of hours the bakery was open each day
C. the number of hours each employee worked this month
D. the number of customers who visited the bakery this week
E. the total number of hours the employees have worked this year

Answers

The two quantities needed to determine the total amount of money Monte paid his employees this month are:

A. the hourly salary of each employee

C. the number of hours each employee worked this month.

How the total salary for the month is determined?

With four employees at the Monte Bakery, who earn a different hourly wages, to work out the total salary for the month, Monte needs to multiply the hourly wage of each employee by the total number of hours each worked.

Thus, in this mathematical operation, we first use multiplication and then add each individual employee's total salary to sum the total salary for the month.

Learn more about mathematical operations for determining the total at https://brainly.com/question/4721701

#SPJ1


Help me on the questions please and thank you.

Answers

The lateral surface area of the rectangular prism is 308 units² and the total surface area is 398 units²

What is the lateral and total surface area of the rectangular prism

The formula of lateral surface area of the rectangular prism is given as;

LSA = 2(l +b)h

l = length b = breath or widthh = height

substituting the values into the formula;

LSA = 2(5 + 9) * 11

LSA = 308 units²

The total surface area is given by

TSA = 2(lb + bh + lh)

TSA = 2[(5*9) + (9*11) + (11*5)

TSA = 398 units²

Learn more on lateral surface area here;

https://brainly.com/question/15777841

#SPJ1

Write a quadratic function f whose zeros are -2 and 12.

f(x) = 0

Answers

Answer:

[tex]f(x) = (x + 2)(x - 12)[/tex]

[tex]f(x) = {x}^{2} - 10x - 24[/tex]

what is true about the slope/derivative of a vertical tangent?

Answers

The slope and derivative of a vertical tangent are both undefined due to the vertical nature of the tangent line.

The slope/derivative of a vertical tangent.
A vertical tangent occurs when a curve has a point where its tangent line is vertical.

In terms of the slope and derivative, the following is true about the vertical tangent:
Slope:

The slope of a vertical tangent is undefined because the vertical line has no run (change in x). Slope is calculated as the rise (change in y) divided by the run (change in x), and division by zero is not possible.
Derivative:

The derivative of a function at a point represents the slope of the tangent line to the curve at that point.

When the tangent is vertical, the derivative is also undefined because the slope is undefined.

For similar question on tangent.

https://brainly.com/question/1533811

#SPJ11

Here is your sketch of a cylinder with a radius of 3 and
a height of 10.
Calculate the volume of the cylinder.
Express your answer in terms of it.

Answers

The volume of the cylinder is 90π cubic units.

To calculate the volume of a cylinder with a radius of 3 and a height of 10, you can use the following formula:

Volume = π × r² × h

Where:

- Volume represents the volume of the cylinder,

- π (pi) is a mathematical constant approximately equal to 3.14159,

- r is the radius of the cylinder (in this case, 3), and

- h is the height of the cylinder (in this case, 10).

Now, plug in the given values:

Volume = π × (3)² × 10

Simplify the expression:

Volume = π × 9 × 10

Volume = 90π

So, the volume of the cylinder is 90π cubic units.

Learn more about volume here,

https://brainly.com/question/218706

#SPJ1

An airplane flight has 228 seats. The probability that a person who buys a ticket actually goes on that flight is about 95%. If the airline wants to fill all the seats on the flight, how many tickets should it sell?
345 tickets
240 tickets
2400 tickets
217 tickets

Answers

Answer:

240 tickets sold

Step-by-step explanation:

first, you would set up the proportion by writing [tex]\frac{228}{95} =\frac{x}{100}[/tex]

with "x" being the number of seats that needs to be sold

then, you would cross multiply

and you get

[tex]95x=228*100[/tex]    next, you simplify to get the inequality [tex]95x=22800[/tex]

then divide both sides by 95

[tex]x=240[/tex]

so 240 tickets to fill all the seats

hope this helps ;)

The center of a circle is at (−2, −7) and its radius is 6.

What is the equation of the circle?

Responses

(x+2)2+(y+7)2=3
open parenthesis x plus 2 close parenthesis squared plus open parenthesis y plus 7 close parenthesis squared equals 3

(x+2)2+(y+7)2=36
open parenthesis x plus 2 close parenthesis squared plus open parenthesis y plus 7 close parenthesis squared equals 36

(x−2)2+(y−7)2=3
open parenthesis x minus 2 close parenthesis squared plus open parenthesis y minus 7 close parenthesis squared equals 3

(x−2)2+(y−7)2=36

Answers

option B is correct: [tex](x+2)^2+(y+7)^2=36[/tex]

The general equation of the circle is given by:

[tex](x-h)^2+(y-k)^2=r^2[/tex]              

where,

(h, k) is the center of the circle and r is the radius of the circle.

As per the statement:

The center of a circle is at (−2, −7) and its radius is 6.

[tex]\implies (h, k) = (-2, -7)[/tex] and [tex]r = 6[/tex] units

Substitute these we have:

[tex](x-(-2))^2+(y-(-7))^2=6^2[/tex]

[tex]\implies(x+2)^2+(y+7)^2=36[/tex]

Therefore, the equation of circle is, [tex]\bold{(x+2)^2+(y+7)^2=36}[/tex]

Airplane tickets to Fairbanks Alaska, will cost $958 each. Airplane tickets to Vancouver but I will call $734. How much can a four members of the Harrison family save on airfare by vacationing in Vancouver?

Answers

A four members of the Harrison family save  $896 on airfare by vacationing in Vancouver.

Here, the airplane tickets to Fairbanks Alaska will cost $958 each.

⇒ c₁ = $958

where c₁ represents the airfare to Fairbanks Alaska

and the airplane tickets to Vancouver Canada will cost $734.

⇒ c₂ = $734

Let us assume that s represents the amount of savings per ticket.

⇒ s = c₁ - c₂

⇒ s = 958 - 734

⇒ s = $224

Since there are four members in the family.

So using unitary method the total savings would be,

⇒ t = 4 × s

⇒ t = 4 × 224

⇒ t = $896

Therefore, the total savings =  $896

Learn more about the unitary method here;

https://brainly.com/question/28276953

#SPJ4

The complete question is:

Airplane tickets to Fairbanks Alaska will cost $958 each. Airplane tickets to Vancouver Canada will cost $734. How much can the four members of a Harrison family save on airfare by vacationing in Vancouver?

what is the maximum number of mail swaps bagelbot can perform between officials from a and officials from b? assume every official regularly receives a lot of mail, and bagelbot can redirect any of it anywhere at any time. (i.e., what is the maximum number of edges possible for a bipartite graph between a and b?)

Answers

The maximum number of mail swaps that Bagelbot can perform between officials from A and officials from B can be 150 .

It can be determined by calculating the maximum number of edges possible for a bipartite graph between A and B. A bipartite graph is a graph in which the vertices can be divided into two disjoint sets such that every edge connects a vertex in one set to a vertex in the other set.

In this case, the officials from A and officials from B represent the two disjoint sets.

The maximum number of edges in a bipartite graph between A and B can be calculated as the product of the number of officials in A and the number of officials in B. Therefore, if there are n officials in A and m officials in B, the maximum number of mail swaps that Bagelbot can perform is n x m.

For example, if there are 10 officials in A and 15 officials in B, the maximum number of mail swaps that Bagelbot can perform is 10 x 15 = 150.

This means that Bagelbot can potentially redirect up to 150 pieces of mail between officials from A and officials from B.

Know more about   maximum number   here:

https://brainly.com/question/29795588

#SPJ11

Question 3 out of 8:’sks

Answers

The picture below shows the graph of the inequality of x² ≤ 4

Option B is correct.

What is  inequality in mathematics?

In mathematics, an inequality is described as a relation which makes a non-equal comparison between two numbers or other mathematical expressions which is used most often to compare two numbers on the number line by their size.

A graph of inequality denotes that the variables is the set of points that represents all solutions to the inequality.

A linear inequality divides the coordinate plane into two halves by a boundary line where one half represents the solutions of the inequality.

Therefore, the correct option is B. x² ≤ 4

Learn more about inequality at:

brainly.com/question/25275758

#SPJ1

example of solving related rate problem:Water leaking onto the floor creates a circular pool with an area that increases at a rate of 3cm^2 / min. How fast is the radius of the pool changing at the moment when the radius is 10 cm?

Answers

The radius of the pool is changing at a rate of 3 / (20π) cm/min when the radius is 10 cm.

To solve this problem, we'll use the given information and the area formula for a circle.
Write the formula for the area of a circle:

[tex]A = \pi r^2[/tex],

where A is the area and r is the radius.
Differentiate both sides of the equation with respect to time (t): [tex]dA/dt = d(\pi r^2)/dt.[/tex]
Apply the chain rule on the right side: [tex]dA/dt = 2\pi r(dr/dt),[/tex]

where dr/dt is the rate of change of the radius.
Plug in the given values:

dA/dt = 3 cm²/min (given) and r = 10 cm (given at the moment of interest).
Solve for dr/dt:
3 = 2π(10)(dr/dt).

Isolate dr/dt:
dr/dt = 3 / (20π)
Simplify:
dr/dt = 1 / (20π/3) = 3 / (20π).

For similar question on radius.

https://brainly.com/question/28308685\\

#SPJ11

May you please go over step by step of how to do this

Answers

The missing lengths of a geometric system are listed below: AC = 114, BC = 96, BE = 3

How to determine missing lengths of two similar triangles

In this problem we find a geometric system formed by two similar triangles. Two triangles are similar if every pair of corresponding sides have the same proportion. Then, the system is described by this proportion formula:

AB / AC = AE / AD = BE / CD

18 / AC = 15 / 95 = BE / 19

Then,

18 / AC = 15 / 95

AC = 18 × (95 / 15)

AC = 114

15 / 95 = BE / 19

BE = 19 × (15 / 95)

BE = 3

Now, we determine the length of side BC:

BC = AC - AB

BC = 114 - 18

BC = 96

To learn more on similar triangles: https://brainly.com/question/30447583

#SPJ1

Divide. Give the Quotient and Remainder. 520 / 22

Answers

Answer:

Quotient=23 and remainder=14

QUICK QUICKQUICK QUICKQUICK QUICKQUICK QUICKQUICK QUICKQUICK QUICKQUICK QUICKQUICK QUICKQUICK QUICKQUICK QUICKQUICK QUICKQUICK QUICKQUICK QUICKQUICK QUICK

Answers

Answer:

  15.0 in

Step-by-step explanation:

You want the height of a cone with radius 8 in and slant height 17 in.

Pythagorean theorem

The right triangle shown in the figure has height h, base x = 8 in, and slant height y = 17 in. The Pythagorean theorem tells you the relationship between these side lengths:

  x² +h² = y²

  8² +h² = 17²

  h² = 17² -8² = 289 -64 = 225

  h = √225 = 15.0

The height of the cone is 15.0 inches.

__

Additional comment

A triple of 3 integers that form the sides of a right triangle is called a "Pythagorean triple." The triple in this problem is one of those: {8, 15, 17}. Other Pythagorean triples you will often see in algebra, trig, and geometry problems are {3, 4, 5}, {5, 12, 13}, {7, 24, 25}, {9, 40, 41}. You will also see multiples of these, for example 3×{3, 4, 5} = {9, 12, 15}.

It can be worthwhile to remember some of these. For this problem, recognizing 8 and 17 as part of the triple {8, 15, 17} means you can write down the answer with no further work.

If a reduced echelon matrix T(x) = 0 has a row of [ 0 . . 0 | 0] or [0 . . .0 | b] , where b =/= 0, it's considered one to one.
a. true
b. false

Answers

The correct answer is false. A reduced echelon matrix [tex]T(x) = 0[/tex] with a row of [tex][0 . . 0 | 0] or [0 . . . 0 | b][/tex], where[tex]b \neq 0[/tex], is not considered one-to-one.

Let's first define what it means for a matrix to be one-to-one.

A matrix is said to be one-to-one if each column of the matrix is linearly independent.

This means that for any given input, there is only one possible output.
Now, let's consider the reduced echelon matrix T(x) = 0. If this matrix has a row of [tex][0 . . 0 | 0] or [0 . . . 0 | b][/tex], b ≠ 0, it means that one of the variables in the system of equations represented by the matrix is a free variable.

This free v[tex]T(x) = 0.[/tex]ariable can take on any value, and the other variables will adjust accordingly.
If the reduced echelon matrix is [tex][1 0 | 2; 0 0 | 0],[/tex] the system of equations would be [tex]x = 2[/tex] and y can be any value.

This means that the matrix is not one-to-one, because there are multiple outputs for the same input.
the correct answer is false.

A reduced echelon matrix[tex]T(x) = 0[/tex] with a row of [tex][0 . . 0 | 0] or [0 . . . 0 | b][/tex], where [tex]b \neq 0[/tex], is not considered one-to-one.

For similar questions on matrix

https://brainly.com/question/1279486

#SPJ11

Quadrillion ABCD is dilated to create 2 images, as shown

Answers

Quadrilateral ABCD can be dilated by a scale factor 1.25 to create a smaller image and by a scale factor 1.5 to create a bigger image.

How to calculate the scale factor of the given quadrilaterals?

To calculate the scale factor of a given shape for its dilation or reduction, the formula that should be used is given as follows:

Scale factor = bigger dimension/smaller dimension.

To create a smaller image;

bigger dimension length AB = 22in

smaller dimension length AB = 17.6in

Scale factor = 22/17.6 = 1.25

To create a bigger image:

Bigger dimension length = 33in

Smaller dimension length = 22in

scale factor = 33/22 = 1.5

Learn more about scale factor here:

https://brainly.com/question/28339205

#SPJ1

A sign on the roadway at the top of a mountain indicates that for the next 4 miles the grade is 9.5° (see figure). Find the change in elevation for a car descending the 4-mile stretch. (Round your answer to two decimal places.)

Answers

Step-by-step explanation:

We can use trigonometry to solve this problem. The grade of the road is given as an angle of 9.5°. This means that for every 1 unit of horizontal distance traveled, the elevation changes by the tangent of 9.5°.

We want to find the change in elevation for a car descending the 4-mile stretch. To do this, we can multiply the length of the road by the tangent of the grade angle:

Change in elevation = 4 miles x tan(9.5°)

Using a calculator, we can find that tan(9.5°) = 0.1664. Substituting this value into the equation, we get:

Change in elevation = 4 miles x 0.1664 ≈ 0.67 milesTo convert this to feet, we can multiply by the number of feet in a mile:

Change in elevation = 0.67 miles x 5,280 feet/mile ≈ 3,539 feetTherefore, the change in elevation for a car descending the 4-mile stretch is approximately 3,539 feet.

The power of -1 to a function, or ((f^-1)(x)), means...

Answers

The power of -1 to a function, or [tex]((f^{-1})(x)),[/tex]represents the inverse function of f. In other words, if f(x) produces a certain output y, then the inverse function[tex]f^{-1}(y)[/tex]will produce the input x. It is important to note that not all functions have an inverse, and if they do, they may only be valid for certain inputs and outputs.

In mathematics, the inverse function of a function f (also called the inverse of f) is a function that undoes the operation of f1. The inverse of f exists if and only if f is bijective.

[tex]((f^{-1})(x)),[/tex]

The inverse function of f is denoted by. For a function, its inverse admits an explicit description: it sends each element to the unique element such that f(x) = y1.

learn more about inverse function

https://brainly.com/question/2541698

#SPJ11

1. Find volume, show work round to 2 decimal places

*cylinder*

Answers

in this case since it's a cylinder always have to do is multiply the two given measurements dancing that she gets all you need to do is run it off to the nearest two decimal places when you say to the nearest two decimal places while trying to say they are two numbers after the coma

Find the surface area. Round your answer to the nearest tenth if necessary. Just enter the number, not the unit.

Answers

The total surface area of the prism is  177.2 m².

What is the surface area of the prism?

The total surface area of the prism is calculated by applying the formula for total surface area of prism.

S.A = bh + (s₁ + s₂ + s₃)L

where;

b is the base of the triangleh is the height of the triangles₁ is the first triangular faces₂ is the second triangular faces₃ is the third triangular faceL is the length of the prism

The surface area of the prism is calculated as;

S.A = 4 m (3 m) + (3.8 m + 4 m + 4 m) x 14 m

S.A = 177.2 m²

Thus, the surface area of the prism is calculated using the formula for surface of right prism.

Learn more about surface area of prism here: brainly.com/question/30337697

#SPJ1

Graph a line with a slope of 3/4 that contains the point (2,-3)

Answers

Step-by-step explanation:

[tex] - 3 = \frac{3}{4} (2) + b[/tex]

[tex] - \frac{6}{2} = \frac{3}{2} + b[/tex]

[tex]b = - \frac{9}{2} [/tex]

[tex]y = \frac{3}{4} x - \frac{9}{2} [/tex]

Alternately, starting at (2, -3), go up 3 units, then right 4 units, to (6, 0). Draw a line that goes through these points.

Jenny received a $1900 bonus. She decided to invest it in a 5-year certificate of deposit (CD) with an annual interest rate of 1.47% compounded quarterly.
Answer the questions below. Do not round any intermediate computations, and round your final answers to the nearest cent. If necessary, refer to the
list of financial formulas.
(a) Assuming no withdrawals are made, how much money is in Jenny's account
after 5 years?
$
(b) How much interest is earned on Jenny's investment after 5 years?
$

Answers

After Jenny invested the $1900 in a 5-year CD,

The money Jenny will have in her account after 5-years is $2400The interest she earned after 5-years is $500

A) Given principal money (P) = $1900

Rate of interest (r) = 1.47% / 100 = 0.047

number of times interest compounded per year (n) = 4

number of years (t) = 5

Amount Jenny will have after 5-years (A) = [tex]p(1+r/n)^{nt}[/tex]

A = [tex]1900(1+0.047/4)^{4*5}[/tex]

A = [tex]1900(1 + 0.01175)^{20}[/tex]

A = [tex]1900(1.01175)^{20}[/tex]

A = 1900(1.2631) ≅ $2400

B) Interest earned after 5 years = $2400 - $1900 = $500.

To know more about interest problems,

https://brainly.com/question/25720319

#SPJ1

Depreciation by Three Methods; Partial Years Perdue Company purchased equipment on April 1 for $41,550. The equipment was expected to have a useful life of three years, or 5,700 operating hours, and a residual value of $1,650. The equipment was used for 1,000 hours during Year 1, 2,000 hours in Year 2, 1,700 hours in Year 3, and 1,000 hours in Year 4. Required: Determine the amount of depreciation expense for the years ended December 31, Year 1, Year 2, Year 3, and Year 4, by (a) the straight-line method, (b) the units-of-activity method, and (c) the double-declining-balance method. Note: Round all final values for each depreciation method and each year to the nearest whole dollar

Answers

a) The depreciation expenses for Year 1 to Year 4 using the straight-line method are $2,632, $5,263, $4,211, and $2,632, respectively.

b) Using the units-of-activity method, the depreciation expenses are $7,000, $14,000, $11,900, and $7,000, respectively.

c) Using the double-declining-balance method, the depreciation expenses are $4,658, $11,172, $7,302, and $2,640, respectively.

a) To calculate the depreciation expense for each year using the straight-line method, the company needs to first determine the depreciable cost of the equipment. This is the cost of the equipment minus its estimated residual value.

The depreciable cost is $41,550 - $1,650 = $39,900.

To calculate the annual depreciation expense using the straight-line method, divide the depreciable cost by the useful life of the equipment, which is three years.

Year 1: ($39,900 ÷ 3) × (1,000 ÷ 5,700) = $2,632

Year 2: ($39,900 ÷ 3) × (2,000 ÷ 5,700) = $5,263

Year 3: ($39,900 ÷ 3) × (1,700 ÷ 5,700) = $4,211

Year 4: ($39,900 ÷ 3) × (1,000 ÷ 5,700) = $2,632

b) To calculate the depreciation expense using the units-of-activity method, the company needs to first determine the depreciation rate per unit of activity. This is the depreciable cost of the equipment divided by its total estimated hours of use.

The depreciation rate per unit of activity is $39,900 ÷ 5,700 hours = $7 per hour.

Year 1: $7 × 1,000 hours = $7,000

Year 2: $7 × 2,000 hours = $14,000

Year 3: $7 × 1,700 hours = $11,900

Year 4: $7 × 1,000 hours = $7,000

c) To calculate the depreciation expense using the double-declining-balance method, the company needs to first determine the straight-line depreciation rate. This is the depreciable cost of the equipment divided by its useful life.

The straight-line depreciation rate is $39,900 ÷ 3 years = $13,300 per year.

The double-declining-balance depreciation rate is twice the straight-line rate, or $13,300 × 2 = $26,600.

Year 1: $26,600 × (1,000 ÷ 5,700) = $4,658

Year 2: ($39,900 - $4,658) × (2,000 ÷ 5,700) = $11,172

Year 3: ($39,900 - $4,658 - $11,172) × (1,700 ÷ 5,700) = $7,302

Year 4: ($39,900 - $4,658 - $11,172 - $7,302) × (1,000 ÷ 5,700) = $2,640

To learn more about costs click on,

https://brainly.com/question/15224550

#SPJ4

William is creating a film for a school project using a digital video camera.

William transfers the videos to a computer for editing.

(i) The computer has 1GB of storage free.

Calculate the number of videos that could be stored on the computer if each video was 100MB in size.

Show your working.

(ii) A program needs to calculate the size of files in bytes. The program must:

· Ask the user to input a file size in megabytes
· calculate and output the number of bytes this represents in a user friendly format.
(e.g. "There are 5242880 bytes in 5MB").

Write an algorithm using pseudocode to calculate the number of bytes in a given number of megabytes.

Answers

(i) To calculate the number of videos that could be stored on the computer:

1GB = 1000MB (since 1GB = 1024MB, but we'll use the simpler 1000MB approximation here)

Number of videos = (1GB free space) / (100MB per video)
Number of videos = (1000MB) / (100MB per video)
Number of videos = 10 videos

Therefore, if each video is 100MB in size, William can store a maximum of 10 videos on the computer.

(ii) Algorithm to calculate the number of bytes in a given number of megabytes:

1. Ask the user to input the file size in megabytes.
2. Read the input and store it in a variable, let's call it "megabytes".
3. Calculate the number of bytes by multiplying "megabytes" by 1,048,576 (this is the number of bytes in a megabyte).
4. Output the result in a user-friendly format, such as: "There are [number of bytes] bytes in [megabytes] MB."

Here's the pseudocode:

```
INPUT megabytes
bytes = megabytes * 1048576
OUTPUT "There are " + bytes + " bytes in " + megabytes + " MB."
```

Answer:

(i) Calculation:

1 GB = 1000 MB

Number of videos that could be stored = (1 GB / 100 MB) = 10

Therefore, William can store 10 videos on his computer if each video is 100MB in size.

(ii) Algorithm using pseudocode:

Ask the user to input the file size in megabytes.

Multiply the file size by 1,000,000 to convert it to bytes.

Print the result in a user-friendly format by dividing the bytes by 1,048,576 and rounding to two decimal places, and adding "MB" and "bytes" to the output.

Pseudocode:

Input file_size_in_mb

Set bytes = file_size_in_mb * 1000000

Set result = round((bytes / 1048576), 2)

Output "There are " + result + " MB " + bytes + " bytes".

2. A pair of jeans is on sale for 25% off the original price. Which expression represents the sale
price? If the original price of the jeans is $40, evaluate the expression to find the sale price.
Os=p-0.25p; $30
Os=p-25; $15
Os=p-0.25p; $10
Os p-0.25; $39.75

Answers

Answer: s = p - 0.25p; $30

Step-by-step explanation:

      First, we need to know that a percent divided by 100 becomes a decimal.

          25% / 100 = 0.25.

      Next, we will use this equation because it takes 25% off of the original price (p) by subtracting 0.25p from p.

          s = p - 0.25p

      Lastly, we will substitute 40 in for 0 and solve.

          s = ($40) - 0.25($40)

          s = $30

Other Questions
Uncertainty refers to situations in which the probabilities of the various possible outcomes are known, whereas risk refers to situations in which no objective probability data are available.True or False? If you are in danger of a potential head on collision, what should you do? A good brewery uses ________ times more water than ends up in the beer Which proportion can be used to find 180% of 45? Chapter 10: Quadratic Relations and Conic Sections Answers corporations will want to be cautious about what is posted to wikis since wikis can be accessed by anyone The performing forces in the first movement of Symphony No. 5 in C Minor consist of: T/F: "Contained in" applies when a properly marked source document contains classified information that has been extracted word-for-work and placed into the new document. What is NOT a dimension of the V-R-I-O framework?A. InimitableB. RoutinelyC. OrganizedD. Valuable Which drug and dose are recommended for the management of a pt. in refractory V-FIB? If Bob cleans his room, then Bob gets ice cream.Bob does not get ice cream.Therefore, Bob does not clean his room. In the book Eragon Chapters Hall of the Mountain through The Mourning SageWhat does Murtagh tell Eragon happened after the Shade was killed?(a) A lightning bolt fell onto the remaining armies(b) The Twins were able to defeat the rest(c) Elf warriors arrived and helped(d) The Urgals turned on each other a(n) lock exists when concurrent transactions are granted read access on the basis of a common lock. question 5 options: a) shared b) two-phase c) exclusive d) binary How was the power of the federal government both expanded and challenged during the Jacksonian Era? Be sure to explain at least two ways federal power was expanded and two ways it was challenged Home Products Company and House & Yard, Inc., use the mark "Good Housekeeping Seal of Approval" to certify the quality of their products. Home Products and House & Yard are not in business together and do not own this mark. This mark is:a. a certification markb. a collective markc. a service markd. trade dress There are three angles measured in degrees of a triangle where x is the largest angle, y is the middle angle, and z is the smallest angle. The measure of the largest angle is 40 degrees less than twice the sum of the measure of the other two angles. The measure of the largest angle is twice the smallest angle plus 10 degrees. The sum of the angles of a triangle can be written as x + y + z = 180. What are the other equations that represent this system of equations? Select each correct answer.a.) x=2y+10b.) x=2z+10c.) x=40-2(y+z)d.) x=2z+y-40e.) 2(y+z)-40 Question 20 Marks: 1 The peak seasons for rats to breed in temperate zones areChoose one answer. a. spring and fall b. summer and winter c. winter and spring d. fall and summer 8a. what type of tissue is this? (include subtype) 8b. what type of cell secretes the matrix for this tissue? What is the recommended initial therapy for a patient with stable narrow-complex tachycardia, after establishing an IV and acquiring a 12-lead ecg?a.Adenosine b.B-blockersc.Cardioversiond.Vagal maneuvers 3. What report shows what mobile devices were used to view a website?A. The Exit Pages report under "Site Content"B. The Landing Page report under "Site Content"C. The Engagement report under "Behavior"D. The Devices report under "Mobile"